LSAT and Law School Admissions Forum

Get expert LSAT preparation and law school admissions advice from PowerScore Test Preparation.

 Administrator
PowerScore Staff
  • PowerScore Staff
  • Posts: 8950
  • Joined: Feb 02, 2011
|
#65994
Please post your questions below!
 g_lawyered
  • Posts: 213
  • Joined: Sep 14, 2020
|
#95272
Hi P.S.,
I was able to find support for correct answer E in the last lines of paragraph 4. However, I had answer choice A as contender. What makes answer A incorrect?
Thanks in advance
 Rachael Wilkenfeld
PowerScore Staff
  • PowerScore Staff
  • Posts: 1419
  • Joined: Dec 15, 2011
|
#95297
Hi g-lawyered,

Answer choice (A) is a very strongly worded answer choice, meaning that we would need very strong language to support it in the stimulus.

"Any further decline WOULD NOT be caused by humans" is an extreme statement. Let's say that 3 fish die due to an ocean spill. The passage wouldn't say that's impossible. But answer choice (A) would! That shows us that answer choice (A) doesn't have support in the passage.

Remember most of the questions in RC are in the prove family. Strong language in these answer choices is dangerous and difficult to support. I'm not saying rule things out just because there's strong language. But you should certainly give something like answer choice (A) a healthy dose of side eye.

Hope that helps!
 g_lawyered
  • Posts: 213
  • Joined: Sep 14, 2020
|
#95328
Hi Rachael,
I understand what you mean about A having extreme language. Can answer A also be disprove because of what's stated in paragraph 3 " Even more important, intensive farming of many species of fish requires large inputs of fish meal and fish oil in order to supply fatty acids that vegetable matter lacks or essential amino acids that it inadequately supplies, like lysine and methionine.". This intense fish farming is said to increase- support in paragraph 1 "Fish farming produces a quarter of all fish and shellfish eaten by humans, and, as global population increases, fish farming will probably become even more important in supplying human protein needs.". Does this mean that because humans are the main contributor to fish farming (which decreases ocean fishery stocks), that further declines occurs BECAUSE of humans? Does this disprove and make A incorrect answer? Or is it only because of the extreme language that it uses (as in your explanation)?
I'm trying to improve on recognizing what makes incorrect answer choices wrong.
Thanks in advance
 Adam Tyson
PowerScore Staff
  • PowerScore Staff
  • Posts: 5400
  • Joined: Apr 14, 2011
|
#95342
I tend to approach answers like this from a different angle, g_lawyered. It's not our job to prove that the answer is false; it's our job to select the one answer that is supported by the passage, and that means the wrong answers are not supported. That means an answer that might be true, but which has no support, would be a wrong answer. It's about NOT finding support, rather than finding something that disproves it. So, for example, if an answer said "some consumers prefer the texture of farmed cod to that of wild-caught cod," the passage wouldn't prove that was false, but it would be a wrong answer because there is no support for it in the passage. It's the lack of support that makes it wrong, not the evidence against it.

Answer A is wrong because there is no support for it. There's nothing there to suggest that anything other than humans could cause further declines. Maybe something other than human activity would cause further declines, and maybe not - there's just nothing in the text to support this position, and that's why it's a wrong answer.
 g_lawyered
  • Posts: 213
  • Joined: Sep 14, 2020
|
#95363
Hi Adam,
Thank so much for explaining your technique for this question type! It really helped my outlook on it. :)
 arjanlion@gmail.com
  • Posts: 3
  • Joined: Jul 22, 2021
|
#96940
Hey,

I chose B due to what is stated about salmon, tilapia, and channel catfish in the last paragraph. I see now that E is the best answer and don't know how I overlooked it, but is B incorrect due to the word best being an overemphasis? Introducing a farmer variety into the market may reduce the price of wild fish, but we do not know if it is the best way to do so.
 Luke Haqq
PowerScore Staff
  • PowerScore Staff
  • Posts: 938
  • Joined: Apr 26, 2012
|
#97021
Hi arjanlion!

Yes, your intuition is right that a problem with answer choice (B) is that it is too strong.

As you note, (B) refers to the "best way to reduce the price." You're also right to look to the final paragraph in considering this answer choice. The most pertinent information is at lines 50-51: "For example, increasing the farm production of fish like salmon can reduce prices."

This material, however, isn't making a claim about the "best" means for reducing prices. Rather, it's a statement only that a certain route "can reduce prices." It's possible that this is the best means for reducing price but we do not know one way or the other from the passage. Since this is a must-be-true question, we can therefore eliminate (B) for that reason alone.

In addition, it's also worth noting that the claim at lines 50-51 is only about salmon. Answer choice (B), by contrast, is about "wild-caught fish" generally. Even if we supposed those lines did make a claim about the best means, it would only be about what is best with respect to salmon, rather than with respect to wild-caught fish generally. This is another reason that would make answer choice (B) incorrect.

Hope that helps!

Get the most out of your LSAT Prep Plus subscription.

Analyze and track your performance with our Testing and Analytics Package.